LSAT and Law School Admissions Forum

Get expert LSAT preparation and law school admissions advice from PowerScore Test Preparation.

User avatar
 Dave Killoran
PowerScore Staff
  • PowerScore Staff
  • Posts: 5853
  • Joined: Mar 25, 2011
|
#92496
Complete Question Explanation
(The complete setup for this game can be found here: lsat/viewtopic.php?f=272&t=8596)

The correct answer choice is (B)

If K is at Souderton, then only Template #2 applies:

G4-Q24-d1.png

Accordingly, answer choice (B) is correct.
You do not have the required permissions to view the files attached to this post.
 Hope2CrushLSAT
  • Posts: 3
  • Joined: Aug 20, 2017
|
#41838
Question 24 has 2 correct answers. B. and E.

If E. is incorrect please let me know how.
User avatar
 Stephanie Oswalt
PowerScore Staff
  • PowerScore Staff
  • Posts: 811
  • Joined: Jan 11, 2016
|
#41881
Hope2CrushLSAT wrote:Question 24 has 2 correct answers. B. and E.

If E. is incorrect please let me know how.
Hi Hope2CrushLSAT!

Thanks for the question! The correct answer for #24- "If Kudrow is at Souderton, then which one of the following must be true?" is B. An instructor will explain why E is incorrect below.

Thanks!
 Luke Haqq
PowerScore Staff
  • PowerScore Staff
  • Posts: 747
  • Joined: Apr 26, 2012
|
#41893
Hi Hope2CrushLSAT!

On this must-be-true question, to see why (E) cannot be the right answer but why (B) must be true, I think it's important to emphasize the binary element in this game.

That is, we know that all of the doctors are used once and only once. In addition, we know they're either going too be at only one of two possible places: Souderton (S) or Randsborough (R). The fact that it's either R/S can be powerful if you apply it to contrapositives of the original rules. For example, the first rule "Kudrow is at Randsborough if Juarez is at Souderton" would be diagrammed as:

J(S) :arrow: K(R)

Now, you might draw the contrapositive as follows, which is technically correct (~ means "not"):

~K(R) :arrow: ~J(S)

If K is not in R, then we know K must be in S. So "~K(R)" is the same thing as "K(S)." So we could instead write that as:

K(S) :arrow: J(R)

Similarly N(R) :arrow: O(R) would have a contrapositive of O(S) :arrow: N(S). To answer question #24, using the contrapositives and the original rules, we can construct a chain of what happens if Kudrow is at Souderton:

K(S) :arrow: J(R) :arrow: O(S) :arrow: N(S)

Given this chain, (B) must be true, so it is the correct answer.

If we start with K(S), then we know the placement of four of the doctors: K, J, O, and N. However, we don't know what happens with P and L from that chain. Thus if K(S), then we don't necessarily know P(R), so (E) can't be the correct answer.

Hope that helps!
 Hope2CrushLSAT
  • Posts: 3
  • Joined: Aug 20, 2017
|
#41908
Thanks for your explanation.

I did a chart for this game and the only possible scenarios were the following:

S:KON
R:PLJ

S:PON
R:KL (J) undetermined

S:PJ (N) undetermined
R:OKL

Any other scenario violates one or more of the rules since each the doctors are at exactly one of two clinics. Of these scenarios only the first can satisfy all the rules as well as the conditional rule in question 24. My question is, are we to assume that any question may violate the initial rules of the game, or that there are potentially two correct answers for some questions?
 Francis O'Rourke
PowerScore Staff
  • PowerScore Staff
  • Posts: 471
  • Joined: Mar 10, 2017
|
#41920
Hi Hope2CrushLSAT,

The initial rules of the game apply to all questions. Unless a local rule specifically tells you to ignore or substitute a particular rule from the initial rule set, then each of those rules will apply to the question you are working on. Question #24 does not tell you to do anything like this. It only asks you to add an additional rule into the game while working on this particular question.

As with every question on the LSAT, there is only one correct answer to this question.

I can see why you thought that answer choice (E) must also be true when I look at your list of the different possible scenarios. The problem is that your list does not include every possible placement of doctors to clinic. The possible placement that is important for understanding why answer choices (D) and (E) both could be true is the following:
Souderton ..... Randsborough
.....K ............... L
.....O ............... J
.....N
.....P

I would love to help you out more with understanding this game. so let me know how you came to the conclusion that only those three scenarios were possible.
 Hope2CrushLSAT
  • Posts: 3
  • Joined: Aug 20, 2017
|
#41933
I see what happened. Thank you for your insight.

When approaching this game, would it have been better to make a conditional chain by connecting the rules?
 James Finch
PowerScore Staff
  • PowerScore Staff
  • Posts: 943
  • Joined: Sep 06, 2017
|
#41950
Hi Hope,

Absolutely! In fact this game becomes much simpler when you get the conditional chains together (you always want to create chains based on both the rules and the contrapositives of those rules). I ended up with these chains for the game:

LS :arrow: NR :arrow: OR :arrow: JS :arrow: KR :arrow: PS

and

PR :arrow: KS :arrow: JR :arrow: OS :arrow: NS :arrow: LR

Go back over the questions using these chains; the answers to the questions become really clear! A good setup is the key to efficiently and accurately attacking a logic game, so even if you're feeling some time pressure, make sure to spend the time to work on the setup.

Hope this helps!

Get the most out of your LSAT Prep Plus subscription.

Analyze and track your performance with our Testing and Analytics Package.